Serie di funzioni: differenze tra le versioni

Contenuto cancellato Contenuto aggiunto
Ed088 (discussione | contributi)
perfezionamento esempio
m Bot: Sostituzione automatica (-<br/> +<br />)
Riga 13:
Inoltre, la serie di funzioni <math>\sum_{k=1}^{\infty}f_k</math> si dice '''assolutamente''' convergente in <math>I\,\!</math> se e solo se la serie <math>\sum_{k=1}^{\infty}|f_k|</math> converge puntualmente.
 
Infine, una serie di funzioni di termine generale <math>f_n \,\!</math> si dice totalmente convergente in <math>I\,\!</math> se e solo se:<br />
<math>\exists (M_n)_{n \in \N}, M_n\ge 0 : |f_n(x)|\le M_n, \ \forall x \in I, \forall n \in \N, \ \sum_{k=1}^{\infty}M_k<+\infty</math>
 
Riga 19:
 
===Criteri di Cauchy per le serie di funzioni===
Si vede che, <math>\forall x \in I, \forall n,p \in \N</math>, <math>s_{n+p}(x)-s_n(x)=f_{n+1}(x)+\dots+f_{n+p}(x)\,\!</math>.<br />
Da cui si deducono, a partire dai criteri di Cauchy per le successioni, i seguenti criteri:
 
{{riquadro|border=1px solid blue|contenuto=
La serie di funzioni di termine generale <math>f_n \,\!</math> converge '''puntualmente''' in <math>I\,\!</math> se e solo se:<br />
<math>\forall x \in I, \forall \varepsilon>0, \exists m \in \N : |f_{n+1}(x)+\dots+f_{n+p}(x)|<\varepsilon, \ \forall n>m, \forall p \in \N</math>
}}
 
{{riquadro|border=1px solid blue|contenuto=
La serie di funzioni di termine generale <math>f_n \,\!</math> converge '''uniformemente''' in <math>I\,\!</math> se e solo se:<br />
<math>\forall \varepsilon>0, \exists m \in \N : |f_{n+1}(x)+\dots+f_{n+p}(x)|<\varepsilon, \ \forall x \in I, \forall n>m, \forall p \in \N</math>
}}
Riga 43:
*Sia <math>\sum_{k=1}^{\infty}f_k\,\!</math> una serie di funzioni che converge totalmente in <math>I\,\!</math>. Sia <math>(M_n)_{n \in \N}, M_n\ge 0 : |f_n(x)|\le M_n, \ \forall x \in I, \forall n \in \N, \ \sum_{k=1}^{\infty}M_k<+\infty</math>.
 
Per il criterio di Cauchy relativo alle serie numeriche: <math>\forall \varepsilon>0, \exists m \in \N : M_{n+1}+\dots+M_{n+p}<\varepsilon, \ \forall n>m, \forall p \in \N</math>.<br />
Da ciò segue che, <math>\forall x \in I, \forall n>m, \forall p \in \N</math>: <math>|f_{n+1}(x)+\dots+f_{n+p}(x)|\le|f_{n+1}(x)|+\dots+|f_{n+p}(x)|\le M_{n+1}+\dots+M_{n+p}<\varepsilon</math>.<br />
Quindi è soddisfatto il criterio di Cauchy uniforme sia per la serie <math>\sum_{k=1}^{\infty}|f_k|\,\!</math>, sia per <math>\sum_{k=1}^{\infty}f_k\,\!</math>, ossia le due serie convergono uniformemente, ma per la proposizione precedente, la serie di termine <math>f_n\,\!</math> converge sia assolutamente, sia uniformemente in <math>I\,\!</math>, ossia la tesi. {{endproof}}
 
Riga 53:
}}
====Dimostrazione====
Se la serie di funzioni converge totalmente, allora: <math>\exists (M_n)_{n \in \N}, M_n\ge 0 : |f_n(x)|\le M_n, \ \forall x \in I, \forall n \in \N, \ \sum_{k=1}^{\infty}M_k<+\infty</math>.<br />
Quindi, <math>\forall n \in \N\,\!</math>, <math>f_n\,\!</math> ha un maggiorante che converge. Tuttavia, <math>\sup_{x \in I} |f_n(x)|\le M_n, \forall n \in \N</math>, poiché, per definizione del sup, è il più piccolo dei maggioranti, e quindi la serie del sup converge, per il criterio del confronto tra due serie numeriche. Il viceversa è ovvio.{{endproof}}
 
Riga 66:
===Teorema di integrazione per le serie===
{{riquadro|border=1px solid blue|contenuto=
Se la serie <math>\sum_{k=1}^{\infty}f_k</math> di funzioni integrabili in <math>I\,\!</math> converge uniformemente, allora la serie è integrabile in <math>I\,\!</math>, e vale:<br />
<center><math>\int_{a}^{b} \sum_{k=1}^{\infty}f_k(x)\, dx=\sum_{k=1}^{\infty}\int_{a}^{b} f_k(x)\, dx</math></center>
}}
Riga 73:
===Teorema di derivazione per le serie===
{{riquadro|border=1px solid blue|contenuto=
Se la serie <math>\sum_{k=1}^{\infty}f_k</math> di funzioni derivabili in <math>I\,\!</math> converge in <math>I\,\!</math>, e la serie derivata <math>\sum_{k=1}^{\infty}f'_k</math> converge uniformemente in <math>I\,\!</math>, allora la serie <math>\sum_{k=1}^{\infty}f_k</math> è derivabile in <math>I\,\!</math>, e vale:<br />
<center><math>\left(\sum_{k=1}^{\infty}f_k\right)'=\sum_{k=1}^{\infty}f'_k</math></center>
}}
Riga 88:
}}
====Dimostrazione====
Dato che la serie numerica di termine generale <math>a_n\xi^n\,\!</math> converge, allora la successione <math>(a_n\xi^n)_{n \in \N}\,\!</math> converge a zero, e quindi la successione è limitata, ossia:<br />
<math>\exists M \in \R : \forall n \in \N, |a_n\xi^n|\le M</math>.<br />
Sia <math>\eta \in ]\!-\!|\xi|,|\xi|[\,\!</math>, quindi <math>|\eta|<|\xi|\,\!</math>. Allora, <math>\forall x \in \R : |x|\le|\eta|</math>, <math>|a_nx^n|=|a_n\xi^n|\frac{|x^n|}{|\xi^n|}\le M\left(\frac{|x|}{|\xi|}\right)^n\le M\left(\frac{|\eta|}{|\xi|}\right)^n, \forall n \in \N</math>.<br />
La serie numerica di termine generale <math>M_n=M(|\eta|/|\xi|)^n\,\!</math> è una serie geometrica di ragione strettamente minore di <math>1\,\!</math>, che quindi converge, da cui segue che la serie di potenze converge totalmente in ogni <math>[-\!|\eta|,|\eta|]\subset]\!-\!|\xi|,|\xi|[\,\!</math>, e quindi, scelti <math>a,b \in [-\!|\eta|,|\eta|] : a<b</math>, la serie converge totalmente in ogni <math>[a,b]\subseteq[-\!|\eta|,|\eta|]\subset]\!-\!|\xi|,|\xi|[\,\!</math>, da cui la tesi.
 
Riga 98:
===Teorema 2===
{{riquadro|border=1px solid blue|contenuto=
Sia <math>\rho=\sup X\,\!</math>. Allora <math>\rho\ge 0\,\!</math>, e si hanno tre casi:<br />
*<math>\rho=0 \Leftrightarrow X=\{0\}\,\!</math>
*<math>\rho=+\infty \Leftrightarrow X=\R</math>
Riga 109:
I primi due * sono banalmente dimostrabili.
 
*<br />
<math>\Rightarrow)\,\!</math> Sia <math>x\,\!</math> tale che <math>|x|<\rho\,\!</math>. Allora, per la proprietà dell'estremo superiore, esiste un <math>\xi \in X</math> tale che <math>|x|<\xi\le \rho\,\!</math>. E quindi, per il teorema precedente, la serie converge in <math>x\,\!</math>.<br />
Se, per assurdo, la serie convergesse in un qualche punto <math>\xi\,\!</math> tale che <math>|\xi|>\rho\,\!</math>, allora, per il teorema precedente, la serie convergerebbe, in particolare, in ogni <math>x \in [\rho,|\xi|[</math>, che contraddice il fatto che <math>\rho=\sup X\,\!</math>, da cui l'assurdo.
 
Riga 135:
 
====Dimostrazione====
Per ogni <math>x\neq 0, \lim_{n}\sqrt[n]{|a_nx^n|}=l|x|</math>.<br />
Se <math>l=0\,\!</math>, allora, per il criterio della radice, la serie converge per ogni <math>x \in \R</math>, e quindi <math>\rho=+\infty\,\!</math>.
Se <math>l=+\infty\,\!</math>, allora, per il criterio della radice, la serie converge in solo in <math>0\,\!</math>, e quindi <math>\rho=0\,\!</math>.
Riga 148:
 
====Dimostrazione====
Per ogni <math>x\neq 0, \lim_{n}\left|\frac{a_{n+1}x^{n+1}}{a_nx^n}\right|=l|x|</math>.<br />
Se <math>l=0\,\!</math>, allora, per il criterio del rapporto, la serie converge per ogni <math>x \in \R</math>, e quindi <math>\rho=+\infty\,\!</math>.
Se <math>l=+\infty\,\!</math>, allora, per il criterio del rapporto, la serie converge in solo in <math>0\,\!</math>, e quindi <math>\rho=0\,\!</math>.
Riga 161:
 
====Dimostrazione====
<math>\rho\le\rho')\,\!</math> Supponiamo che la serie di potenze di coefficienti <math>(a_n)_{n \in \N_0}\,\!</math> converga in <math>x_0\,\!</math>. Da ciò segue che la successione (a_nx_0^n)_{n \in \N} converge a zero, e quindi la successione è limitata, ossia <math>\exists L>0 : |a_nx_0^n|\le L \forall n \in \N_0</math>. Allora, <math>\forall |x|<|x_0|, \forall n \in \N_0</math>:<br />
<math>|(n+1)a_{n+1}x^n|=(n+1)\frac{|a_{n+1}x_0^n|}{|x_0^n|}|x^n|=(n+1)\frac{|a_{n+1}x_0^{n+1}|}{|x_0|}\left|\frac{x}{x_0}\right|^n\le \frac{L}{|x_0|}\left|\frac{x}{x_0}\right|^n</math>.<br />
La serie di termine generale <math>|x/x_0|^n\,\!</math> è la serie geometrica di ragione strettamente minore di 1, e quindi convergente. Da ciò segue che la serie di potenze di coefficienti <math>((n+1)a_{n+1})_{n \in \N_0}\,\!</math>, ossia la serie derivata, converge in ogni punto <math>|x|<|x_0|\,\!</math>, da cui segue che <math>\rho\le\rho'\,\!</math>.
 
<math>\rho\ge\rho')\,\!</math> Supponiamo che la serie derivata converge in <math>x_0\,\!</math>. Allora, come prima: <math>\exists M>0 : |(n+1)a_{n+1}x_0^n|\le L \forall n \in \N_0</math>, quindi, <math>\forall |x|<|x_0|, \forall n \in \N</math>:<br />
<math>|a_nx^n|=\frac{1}{n}\frac{|na_nx_0^n|}{|x_0^n|}|x^n|=\frac{1}{n}|na_nx_0^{n-1}||x_0|\left|\frac{x}{x_0}\right|^n\le \frac{M|x_0|}{n}\left|\frac{x}{x_0}\right|^n \le M|x_0|\left|\frac{x}{x_0}\right|^n</math>.<br />
La serie di termine generale <math>|x/x_0|^n\,\!</math> converge, perché è la serie di ragione strettamente minore di 1, da cui segue che la serie di potenze di coefficienti <math>(a_n)_{n \in \N_0}\,\!</math> converge in ogni punto <math>|x|<|x_0|\,\!</math>, da cui segue che <math>\rho\ge\rho'\,\!</math>.{{endproof}}
 
Riga 173:
===Teorema di derivazione e integrazione delle serie di potenze===
{{riquadro|border=1px solid blue|contenuto=
Sia <math>\rho\,\!</math> il raggio di convergenza, supposto non nullo, di una serie di potenze di coefficienti <math>(a_n)_{n \in \N_0}\,\!</math>. Supponiamo che <math>f(x)\,\!</math> la sua somma, ossia:<br />
<math>f(x)=\sum_{k=0}^{\infty}a_kx^k, \ \forall |x|<\rho, \rho>0</math>.<br />
Allora risulta anche: <math>f'(x)=\sum_{k=1}^{\infty}ka_kx^{k-1}, \ \forall |x|<\rho, \ \int_0^t f(t)dt = \sum_{k=0}^{\infty}\frac{a_k}{k+1}x^{k+1}, \ \forall |x|<\rho</math>
}}
Riga 182:
 
===Serie di potenze generalizzato===
Si dice serie di punto iniziale <math>x_0 \in \R</math> e di coefficienti <math>(a_n)_{n \in \N_0}\,\!</math> la serie: <math>\sum_{k=0}^{\infty}a_k(x-x_0)^k</math>.<br />
Ponendo <math>y=x-x_0\,\!</math>, la serie suddetta si riconduce alla serie di punto iniziale 0, da cui si deduce che, se <math>\rho\,\!</math> è il raggio di convergenza della serie di potenze <math>\sum_{k=0}^{\infty}a_kx^k</math>, allora la serie di punto iniziale <math>x_0 \in \R</math> e di coefficienti <math>(a_n)_{n \in \N_0}\,\!</math> converge assolutamente: solo in <math>x_0\,\!</math> se <math>\rho=0\,\!</math>; in <math>\R\,\!</math> se <math>\rho=+\infty\,\!</math>; in ogni punto <math>x\,\!</math> tale che <math>|x-x_0|<\rho\,\!</math> e non converge in ogni punto <math>x\,\!</math> tale che <math>|x-x_0|>\rho\,\!</math>.
 
==Serie di Taylor==
Sia <math>f: ]a,b[ \subseteq \R \to \R, a<b</math>. Sia <math>x_0 \in ]a,b[\,\!</math>.<br />
<math>f\,\!</math> si dice '''sviluppabile in serie di potenze di punto iniziale <math>x_0\,\!</math>''' se esiste una successione numerica <math>(a_n)_{n \in \N_0}\,\!</math> tale che <math>f(x)=\sum_{k=0}^{\infty}a_k(x-x_0)^k, \ \forall x \in ]a,b[</math>
 
===Teorema 1===
{{riquadro|border=1px solid blue|contenuto=
Sia <math>f(x)=\sum_{k=0}^{\infty}a_k(x-x_0)^k, \ \forall |x-x_0|<\rho, \rho>0</math>. Allora <math>f\,\!</math> è indefinitamente derivabile in <math>]x_0-\rho,x_0+\rho[\,\!</math>, e valgono le due uguaglianze:<br />
<math>f^{(m)}(x)=\sum_{k=m}^{\infty}\frac{k!}{(k-m)!}a_k(x-x_0)^{k-m}, \ \forall m \in \N_0, \forall |x-x_0|<\rho</math>, con <math>\frac{k!}{(k-m)!}=k(k-1)\dots(k-m+1), \forall k\ge m</math><br />
<math>f(x)=\sum_{k=0}^{\infty}\frac{f^{(k)}(x_0)}{k!}(x-x_0)^k, \ \forall |x-x_0|<\rho</math>
}}
====Dimostrazione====
Applicando <math>m \in \N</math> volte il teorema di derivazione per le serie di potenze, si ottiene la prima uguaglianza: <math>f^{(m)}(x)=m!a_m+(m+1)!a_{m+1}(x-x_0)+\frac{(m+2)!}{2!}a_{m+2}(x-x_0)^2+\dots+k(k-1)\dots(k-m+1)a_k(x-x_0)^{k-m}+\dots, \ \forall m \in \N_0, \forall |x-x_0|<\rho</math>.<br />
Posto <math>x=x_0\,\!</math>, <math>f^{(m)}(x_0)=m!a_m+0+...+0+...=m!a_m, \ \forall m \in \N_0</math>, da cui <math>a_m=\frac{f^{(m)}(x_0)}{m!}</math>, da cui si ottiene la seconda uguaglianza, ossia la tesi.{{endproof}}
 
<math>f: ]a,b[ \subseteq \R \to \R, a<b</math> indefinitamente derivabile in <math>]a,b[\,\!</math>. <math>f\,\!</math> si dice '''sviluppabile in serie di Taylor di punto iniziale <math>x_0\,\!</math> in <math>]a,b[\,\!</math>''' se: <math>f(x)=\sum_{k=0}^{\infty}\frac{f^{(k)}(x_0)}{k!}(x-x_0)^k, \ \forall x\in ]a,b[</math>
 
La serie al secondo membro si dice '''serie di Taylor della funzione <math>f\,\!</math> di punto iniziale <math>x_0\,\!</math>'''.<br />
La serie di Taylor della funzione <math>f\,\!</math> di punto iniziale <math>0\,\!</math> si dice '''serie di Mac Laurin di <math>f\,\!</math>'''.
 
Riga 211:
}}
====Dimostrazione====
Fissato <math>n \in \N\,\!</math>. Consideriamo il resto <math>n\,\!</math>-esimo di Lagrange della formula di Taylor di <math>f\,\!</math> di punto iniziale un qualunque punto <math>x_0 \in ]a,b[\,\!</math>:<br />
<math>\exists x_1 \in ]a,b[: R_n(x)=f(x)-\sum_{k=0}^{n}\frac{f^{(k)}(x_0)}{k!}(x-x_0)^k=\frac{f^{(n+1)}(x_1)}{(n+1)!}(x-x_0)^{n+1}</math>.<br />
Per ipotesi si ha che: <math>\left|\frac{f^{(n+1)}(x_1)}{(n+1)!}(x-x_0)^{n+1}\right|=\frac{|f^{(n+1)}(x_1)|}{(n+1)!}|x-x_0|^{n+1}\le \frac{ML^{n+1}}{(n+1)!}|x-x_0|^{n+1}, \ \forall n \in \N, \forall x,x_0 \in ]a,b[</math>.
 
La serie di termine generale <math>\frac{ML^n}{n!}|x-x_0|^n</math> converge per il criterio del rapporto: <math>\lim_{n\rightarrow +\infty}\left(\frac{ML^{n+1}}{(n+1)!}|x-x_0|^{n+1}\right)\left(\frac{n!}{ML^n}\frac{1}{|x-x_0|^n}\right)=\lim_{n\rightarrow +\infty}\frac{L|x-x_0|}{n+1}=0</math>.<br />
Da cui segue che: <math>\lim_{n\rightarrow +\infty}\frac{ML^n}{n!}|x-x_0|^n=0</math>, quindi la tesi.{{endproof}}
====Esempio====
Sia <math>f(x)=e^x\,\!</math>. Si sa che <math>f \in C^{\infty}(\R)</math>, e le derivate di <math>f(x)\,\!</math> sono: <math>f^{n}(x)=e^x\,\!</math> , per ogni <math>x \in \R</math> e per ogni <math>n \in \N</math>. <br />
Sia poi <math>R \in \R</math> un numero arbitrario. Dato che la funzione è (strettamente) crescente in <math>\R</math>, allora <math>f^{n}(x)=e^x\le e^R, \ \forall x \le R </math>. <br />
Quindi posto <math> M = e^R\,\!</math> e <math>L =1\,\!</math>, risulta che <math>f^{n}(x)\le ML^n, \ \forall x \le R </math>. <br />
Per il teorema 1 risulta che <math>f\,\!</math> è sviluppabile in serie di Taylor di punto iniziale <math>x_0\,\!</math> in <math>]-\infty,R[\,\!</math>, qualunque sia <math>x_0 \in ]-\infty,R[\,\!</math>. <br />
Dato che <math>R\,\!</math> è stato scelto in maniera arbitraria in <math>\R</math>, allora si può concludere che <math>f\,\!</math> è sviluppabile in serie di Taylor di punto iniziale <math>x_0\,\!</math> in <math>\bigcup_{R \in \R}]-\infty,R[=]-\infty,+\infty[=\R\,\!</math>.
 
===Teorema 2===
{{riquadro|border=1px solid blue|contenuto=
Sia <math>f: ]a,b[ \subseteq \R \to \R, a<b</math> indefinitamente derivabile in <math>]a,b[\,\!</math>. Sia <math>x_0 \in ]a,b[\,\!</math>.<br />
Se <math>f'(x)=\sum_{k=1}^{\infty}\frac{f^{(k)}(x_0)}{(k-1)!}(x-x_0)^{k-1}, \ \forall x\in ]a,b[</math>, allora <math>f\,\!</math> è sviluppabile in serie di Taylor di punto iniziale <math>x_0\,\!</math> in <math>]a,b[\,\!</math>.
}}